Mathematics
Mathematics, 05.12.2020 09:00, angtrevv

Below is a proof showing that the sum of a rational number and an irrational number is an irrational number. Let a be a rational number and b be an irrational number.
Assume that a + b = x and that x is rational.
Then b = x - a = x + (-a).
X + (-a) is rational because
However, it was stated that b is an irrational number. This is a contradiction.
Therefore, the assumption that x is rational in the equation a + b = x must be incorrect, and x should be an irrational
number.
In conclusion, the sum of a rational number and an irrational number is irrational.
Which of the following best completes the proof?
O it is the sum of two rational numbers.
O it is the sum of two irrational numbers.
it represents a non-terminating, non-repeating decimal.
its terms cannot be combined.


Below is a proof showing that the sum of a rational number and an irrational number is an irrationa

answer
Answers: 3

Other questions on the subject: Mathematics

image
Mathematics, 21.06.2019 12:30, queenkimm26
And employee salary increased by $4500 which represents a 15% raise what is the rate of the increase
Answers: 2
image
Mathematics, 21.06.2019 16:30, GalaxyWolf1895
An automated water dispenser fills packets with one liter of water on average, with a standard deviation of 5 milliliter. the manual says that after a year of operation the dispenser should be tested to see if it needs recalibration. a year later a number of filled packets are set aside and measured separately. it is found that the average packet now contains about 0.995 liters. does the dispenser need calibration? explain your answer.
Answers: 2
image
Mathematics, 21.06.2019 17:00, saltytaetae
The tuition costs, c, for a local community college are modeled by c(h) = 250 + 200h, where h represents the number of credit hours taken. the local state university has tuition costs, s, modeled by the function s(h) = 300 + 180h. how many credit hours will a student have to take for the two tuition costs to be equal? round the answer to the nearest tenth of an hour.
Answers: 3
image
Mathematics, 21.06.2019 18:50, BrainzOli7408
If sr is 4.5cm and tr is 3cm, what is the measure in degrees of angle s?
Answers: 2
Do you know the correct answer?
Below is a proof showing that the sum of a rational number and an irrational number is an irrational...

Questions in other subjects: